Matrix with only real eigenvalues

Click For Summary
An nxn matrix with only real eigenvalues can be transformed into an upper triangular form using an orthogonal matrix Q, with eigenvalues positioned along the main diagonal. The proof can be approached through mathematical induction, where the induction step involves finding an orthogonal matrix P that brings the matrix closer to upper triangular form. The discussion emphasizes that gender stereotypes regarding math abilities are unfounded and should not deter anyone from seeking help. Overall, the focus is on the mathematical concept rather than personal feelings or biases. Understanding the process of diagonalization and the properties of orthogonal matrices is key to solving the problem.
Meistro
Messages
5
Reaction score
0
Please help me I am hopelessly lost and don't even know where to start! I guess they're right when they said girls suck at math! It's not fair! :redface:

Let A be an nxn matrix with only real eigenvalues. Prove that there is an orthogonal matrix Q such that (Q^T)AQ is upper triangular with eigenvalues along the main diagonal.


Any of you boys out there help me solve this?
 
Physics news on Phys.org
Meistro said:
Please help me I am hopelessly lost and don't even know where to start! I guess they're right when they said girls suck at math! It's not fair! :redface:

No, "they" are wrong.

Meistro said:
Let A be an nxn matrix with only real eigenvalues. Prove that there is an orthogonal matrix Q such that (Q^T)AQ is upper triangular with eigenvalues along the main diagonal.

You can use induction on n. For a hint on the induction step, you can try to find an orthogonal matrix P where (P^T)AP is 1 step closer to being upper triangular- try to get the first column in the right shape.

Meistro said:
Any of you boys out there help me solve this?

Girls can excel at math just as well as boys, you shouldn't exclude a potential source of aid.
 
Question: A clock's minute hand has length 4 and its hour hand has length 3. What is the distance between the tips at the moment when it is increasing most rapidly?(Putnam Exam Question) Answer: Making assumption that both the hands moves at constant angular velocities, the answer is ## \sqrt{7} .## But don't you think this assumption is somewhat doubtful and wrong?

Similar threads

  • · Replies 18 ·
Replies
18
Views
3K
  • · Replies 1 ·
Replies
1
Views
2K
  • · Replies 4 ·
Replies
4
Views
1K
  • · Replies 2 ·
Replies
2
Views
4K
Replies
5
Views
2K
Replies
9
Views
3K
  • · Replies 14 ·
Replies
14
Views
3K
Replies
7
Views
6K
  • · Replies 2 ·
Replies
2
Views
4K
  • · Replies 2 ·
Replies
2
Views
2K